medical care

This topic has expert replies
Master | Next Rank: 500 Posts
Posts: 171
Joined: Sat Feb 28, 2009 4:59 am
Thanked: 13 times
Followed by:3 members

medical care

by nileshdalvi » Tue Sep 13, 2011 8:45 am
Good medical care for all citizens is a national problem.
Older people are running out of Medicare
benefits and poor people are running out of
Medicaid benefits. Also, it is not uncommon for
people who do have insurance to run up bills above
their insurance limits. But additional government
funding is not the solution. Medical care is
available at free clinics. So, the claim that people
cannot get good medical care is wrong.
That medical care is a problem figures in the discussion
in which one of the following ways?
(A) It is compatible either with accepting the
conclusion or with denying it.
(B) It suggests an alternative perspective to the
one in the argument.
(C) It sets out a problem the argument is designed
to resolve.
(D) It summarizes a position the argument is
directed toward discrediting.
(E) It is required to establish the conclusion.

User avatar
Legendary Member
Posts: 1101
Joined: Fri Jan 28, 2011 7:26 am
Thanked: 47 times
Followed by:13 members
GMAT Score:640

by HSPA » Tue Sep 13, 2011 8:54 am
D looks okay to me
First take: 640 (50M, 27V) - RC needs 300% improvement
Second take: coming soon..
Regards,
HSPA.

User avatar
Legendary Member
Posts: 1309
Joined: Mon Apr 04, 2011 5:34 am
Location: India
Thanked: 310 times
Followed by:123 members
GMAT Score:750

by cans » Tue Sep 13, 2011 8:55 am
D..
Is it valid GMAT problem??
If my post helped you- let me know by pushing the thanks button ;)

Contact me about long distance tutoring!
[email protected]

Cans!!

Master | Next Rank: 500 Posts
Posts: 171
Joined: Sat Feb 28, 2009 4:59 am
Thanked: 13 times
Followed by:3 members

by nileshdalvi » Tue Sep 13, 2011 9:00 am
Even I thought of D, but the OA is A. This looks to be an LSAT Problem present in a book for GMAT. But such problems are good for practice. Just that I found in some cases that the answers are disputable.

User avatar
Legendary Member
Posts: 1101
Joined: Fri Jan 28, 2011 7:26 am
Thanked: 47 times
Followed by:13 members
GMAT Score:640

by HSPA » Tue Sep 13, 2011 9:06 am
May be I havent understood the question : "figures in the discussion" Now I am confused with the meaning for this part..
nileshdalvi wrote:Even I thought of D, but the OA is A. This looks to be an LSAT Problem present in a book for GMAT. But such problems are good for practice. Just that I found in some cases that the answers are disputable.
First take: 640 (50M, 27V) - RC needs 300% improvement
Second take: coming soon..
Regards,
HSPA.

Master | Next Rank: 500 Posts
Posts: 171
Joined: Sat Feb 28, 2009 4:59 am
Thanked: 13 times
Followed by:3 members

by nileshdalvi » Tue Sep 13, 2011 9:30 am
@HSPA: Do you find an interpretation of "figures in the discussion" that suggests that A is the correct answer?

User avatar
Legendary Member
Posts: 1101
Joined: Fri Jan 28, 2011 7:26 am
Thanked: 47 times
Followed by:13 members
GMAT Score:640

by HSPA » Tue Sep 13, 2011 6:14 pm
Figure = presentation.. how is the problem presented

I can see high insurance payout is comman and I can also see free clinic activity. So there is a dual tongue and finally I guess the conclusion is given a little value, it says medical care is going good.

taking premises into account we need to go for A .. but in my opinion using conclusion D it shall be.
nileshdalvi wrote:@HSPA: Do you find an interpretation of "figures in the discussion" that suggests that A is the correct answer?
First take: 640 (50M, 27V) - RC needs 300% improvement
Second take: coming soon..
Regards,
HSPA.

Master | Next Rank: 500 Posts
Posts: 416
Joined: Thu Jul 28, 2011 12:48 am
Thanked: 28 times
Followed by:6 members

by gunjan1208 » Tue Sep 13, 2011 8:32 pm
I was stuck between A and D. But D looked better.....Trapped:)

Master | Next Rank: 500 Posts
Posts: 316
Joined: Sun Aug 21, 2011 6:18 am
Thanked: 16 times
Followed by:6 members

by dhonu121 » Sat Sep 17, 2011 12:15 pm
I am not really sure as to how the OA is A.
The claim that Medical care is a problem, is compatible only with denying the conclusion and not accepting it, as the conclusion clearly states that
So, the claim that people
cannot get good medical care is wrong.

D says just what is there.That the conclusion discredits that medical care is a problem.
Don't know if I am missing anything here.